Finding the instantaneous velocity of a scalar function?

Click For Summary
The discussion revolves around finding the instantaneous velocity of the scalar function f(x, y, z) = xyz at the point (1, 2, 1). Participants express confusion over the problem's formulation, noting that instantaneous velocity is typically associated with vector functions rather than scalar functions. There is a consensus that without additional information, such as the rates of change dx/dt, dy/dt, and dz/dt, the problem cannot be solved as presented. Attempts to use differential notation (dw) are deemed ineffective due to the lack of necessary variables. The conversation highlights the need for clarification on the problem's requirements to proceed.
supermiedos
Messages
62
Reaction score
0

Homework Statement



Find the instantaneous velocity of f(x, y, z) = xyz, at (1, 2, 1)

Homework Equations





The Attempt at a Solution


I think this problem our proffesor gave us wasn't formulated correctly. The only time when we calculated instantaneous velocity was when we had a vectorial function like r(t) = x(t) i + y(t) j + z(t) k.

I tried using dw, but it doesn't work since i don't know dx, dy or dz
 
Physics news on Phys.org
supermiedos said:

Homework Statement



Find the instantaneous velocity of f(x, y, z) = xyz, at (1, 2, 1)

Homework Equations





The Attempt at a Solution


I think this problem our proffesor gave us wasn't formulated correctly. The only time when we calculated instantaneous velocity was when we had a vectorial function like r(t) = x(t) i + y(t) j + z(t) k.

I tried using dw, but it doesn't work since i don't know dx, dy or dz

No, I don't think the phrasing of the question makes much sense.
 
Just to make sure, is there any information that you omitted, like values for dx/dt, dy/dt, and dz/dt? If not, then I concur with Dick.
 
I knew it! No, there's no more information. Only f and the point (1, 2, 1). Thanks!
 
Question: A clock's minute hand has length 4 and its hour hand has length 3. What is the distance between the tips at the moment when it is increasing most rapidly?(Putnam Exam Question) Answer: Making assumption that both the hands moves at constant angular velocities, the answer is ## \sqrt{7} .## But don't you think this assumption is somewhat doubtful and wrong?

Similar threads

  • · Replies 19 ·
Replies
19
Views
9K
  • · Replies 3 ·
Replies
3
Views
2K
  • · Replies 1 ·
Replies
1
Views
1K
  • · Replies 4 ·
Replies
4
Views
2K
  • · Replies 8 ·
Replies
8
Views
4K
  • · Replies 4 ·
Replies
4
Views
2K
Replies
8
Views
2K
  • · Replies 9 ·
Replies
9
Views
1K
  • · Replies 13 ·
Replies
13
Views
2K
Replies
8
Views
2K